Difference between revisions of "2018 AMC 12B Problems/Problem 19"

(Created page with "== Problem == Mary chose an even 4-digit number <math>n</math>. She wrote down all the divisors of <math>n</math> in increasing order from left to right: <math>1,2,..., \frac...")
 
(same problem redirect)
 
Line 1: Line 1:
== Problem ==
+
#REDIRECT[[2018_AMC_10B_Problems/Problem_21]]
 
 
Mary chose an even 4-digit number <math>n</math>. She wrote down all the divisors of <math>n</math> in increasing order from left to right: <math>1,2,..., \frac{n}{2},n</math>. At some moment Mary wrote 323 as a divisor of <math>n</math>. What is the smallest possible value of the next divisor written to the right of 323?
 
 
 
<cmath>\textbf{(A) } 324 \qquad \textbf{(B) } 330 \qquad \textbf{(C) } 340 \qquad \textbf{(D) } 361 \qquad \textbf{(E) } 646</cmath>
 

Latest revision as of 16:01, 16 February 2018